It is a given that to be an intriguing person, one must be able to inspire the perpetual curiosity of others. Constan...

Deniann on November 11, 2013

Diagnostic2, question 13

Can someone please explain this? :)

Reply
Create a free account to read and take part in forum discussions.

Already have an account? log in

Naz on November 15, 2013

This is a Main Point question so we are looking for the answer choice that restates the conclusion of the argument. Since the answer choice will restate the conclusion, it is important to keep in mind that Main Point questions have two criterion for the correct answer: (1) the answer choice must be true and (2) the answer choice must be the main point of the argument.

Remember that the conclusion does not support anything else in the argument. The first sentence starts with "it is a given that." This should be a red flag to cue you into the fact that this is a premise rather than conclusion. "It is given that" starts the evidence the argument works off of to get to the conclusion. The second sentence explains that constantly broadening one's abilities and extending one's intellectual reach will enable one to inspire that curiosity. Why? Well, because of the third sentence: "For such a perpetual expansion of one's mind makes it impossible to be fully comprehended." Thus, the third sentence supports the second sentence, making the second sentence the only one that can be the conclusion. Our correct answer will merely restate the second sentence.

(A) is incorrect because it merely restates the first sentence, which is a premise, not the conclusion.

(B) is CORRECT because it restates the second sentence, which is the conclusion of this argument.

(C) is incorrect because it is restating the third sentence, which is another premise as it supports the conclusion.

(D) is incorrect because the argument never discusses it. We know that constantly broadening one's abilities and extending one's intellectual reach will inspire the perpetual curiosity of others, not the other way around. This is not necessarily true so it cannot be the correct answer.

(E) is incorrect because we do not discuss what causes people to have curiosity in the argument. Again, just like (D), this answer choice is not necessarily true so it cannot be the correct answer.

Hope that was helpful. Let me know if you have any other questions!